1. Trang chủ
  2. » Giáo án - Bài giảng

TH1 b1 HopNV nguyen ly quy nap

22 140 0

Đang tải... (xem toàn văn)

Tài liệu hạn chế xem trước, để xem đầy đủ mời bạn chọn Tải xuống

THÔNG TIN TÀI LIỆU

Phương pháp qui nạp toán học Trần Vinh Hợp - trường THPT chuyên Lê Quý Đôn Quảng Trị Phương pháp chứng minh qui nạp toán học phương pháp chứng minh quan trọng toán học, đặc biệt lĩnh vực toán học rời rạc Phương pháp thường sử dụng toán "chứng minh mệnh đề P (n) với n nguyên ≥ n0 " Có hai dạng thức phương pháp qui nạp, qui nạp (chuẩn) qui nạp mạnh Phương pháp qui nạp Định lí 1.1 (Nguyên lí qui nạp (chuẩn)) Giả sử P (n) mệnh đề chứa biến tự nhiên n định nghĩa với n ≥ n0 Khi đó, mệnh đề P (n) với số nguyên dương n ≥ n0 hai điều sau đồng thời xảy i) Mệnh đề P (n0 ) đúng, ii) Nếu mệnh đề P (n)(n ≥ n0 ) mệnh đề P (n + 1) Chứng minh Xem chứng minh định lí 1.2 Để chứng minh mệnh đề P (n) với n ≥ n0 , ta cần chứng minh hai bước Bước 1: Chứng minh P (n0 ) Bước gọi bước sở thường dễ chứng minh Bước 2: Chứng minh P (n), (với n ≥ n0 ) P (n + 1) Bước gọi bước qui nạp Giả thiết P (n) giả thiết giả định, thường gọi giả thiết qui nạp Ta xét ví dụ sau Ví dụ 1.1 Chứng minh với số nguyên dương n, + + + n = n(n + 1) Lời giải Bước Với n = 1, vế trái đẳng thức cần chứng minh tổng số 1(1 + 1) nguyên dương từ đến 1, nên 1, vế phải = Vậy đẳng thức đề trường hợp n = Bước Giả sử kết luận toán với n ≥ đó, tức + + + n = n(n + 1) (giả thiết qui nạp) Ta cần chứng minh kết luận toán với n + 1, tức chứng minh + + + n + (n + 1) = (n + 1)(n + 2) Thật vậy, ta có + + + n + (n + 1) = (1 + + + n) + n + n(n + 1) + n + (sử dụng giả thiết qui nạp) = (n + 1)(n + 2) = (kết thúc bước 2) Vậy, theo định lí 1.1, ta có điều phải chứng minh Ví dụ 1.2 Với n nguyên dương, tìm cơng thức tường minh cho tổng sau + + + (2n − 1) Chứng minh Khác với ví dụ 1.1, ví dụ ta chưa biết trước kết cho tổng cần tính Do ta cần phải dự đốn để đưa cơng thức tính cho tổng nói đề Thông thường, ta xem xét trường hợp với n nhỏ để đưa dự đoán cho kết toán Chẳng hạn, n = 1, tổng cần tính tổng số lẻ từ đến 1, tổng Khi n = 2, tổng cần tính + = Khi n = 3, + + = Ta để ý thấy = 12 , = 22 , = 32 , vậy, ta đưa dự đoán + + + (2n − 1) = n2 ∀n ≥ Ta chứng minh điều dự đoán Thật vậy, với n = 1, điều cần chứng minh rõ ràng Giả sử ta có + + + (2n − 1) = n2 với n ≥ Khi + + + (2(n + 1) − 1) = (1 + + + (2n − 1)) + (2n + 1) = n2 + 2n + = (n + 1)2 Như vậy, cơng thức dự đốn với n với n + Vậy theo định lí 1.1, ta có + + + (2n − 1) = n2 ∀n ≥ Mấu chốt phương pháp chứng minh qui nạp phải thực bước thứ Và thông thường, để sử dụng giả thiết qui nạp, ta cần tìm cách biểu diễn P (n + 1) theo P (n) Hệ thức biểu diễn P (n + 1) theo P (n) gọi hệ thức truy hồi Trong nhiều tốn, ta khơng tìm hệ thức truy hồi P (n + 1) (chỉ) theo P (n), mà phụ thuộc vào P (n − 1), P (n − 2), , đó, ta cần ngun lí mạnh ngun lí nói định lí 1.1 Ta xét ví dụ sau Ví dụ 1.3 Xét dãy số Fibonacci định nghĩa F0 = F1 = 1, Fn+2 = ∀n ≥ Fn+1 + Fn ∀n ≥ 0(∗) Chứng minh F2n = Fn2 + Fn−1 Phân tích Rõ ràng, với n = 1, khẳng định toán dễ thấy, F2 = = 12 + 12 = F12 + F02 (∗∗) với n ≥ Ta cần chứng minh F2n+2 = Giả sử F2n = Fn2 + Fn−1 2 Fn+1 + Fn Trước hết, ta cần thiết lập hệ thức truy hồi cho F2n+2 theo F2n Áp dụng (*), ta có F2n+2 = F2n+1 + F2n Ta chưa thể áp dụng (**), hệ thức truy hồi có chứa số hạng F2n+1 Nếu tiếp tục sử dụng (*), ta viết tiếp sau F2n+2 = F2n+1 + F2n = 2F2n + F2n−1 , chưa thể áp dụng (**) Ta cần hệ thức truy hồi cho F2n+2 (chỉ) theo số hạng Fk mà có số k chẵn May thay, ta viết F2n−1 = F2n − F2n−2 (suy từ (*)), thu F2n+2 = 2F2n + F2n−1 = 3F2n − F2n−2 2 + Fn−2 Đến đây, ta có F2n−2 = Fn−1 F2n+2 = = = = = = 3F2n − F2n−2 2 3(Fn2 + Fn−1 ) − (Fn−1 + Fn−2 ) 2 2 2(Fn + Fn−1 ) − Fn−2 + Fn (Fn + Fn−1 )2 + (Fn − Fn−1 )2 − Fn−2 + Fn2 2 Fn+1 + Fn−2 − Fn−2 + Fn2 Fn+1 + Fn2 Trên đây, để thu F2n+2 = Fn+1 + Fn2 , ta cần sử dụng đồng thời hai giả thiết 2 qui nạp F2n = Fn2 + Fn−1 F2n−2 = Fn−1 + Fn−2 Do vậy, bước sở, ta phải khẳng định tốn với (ít nhất) hai giá trị n Phương pháp chứng minh phân tích nói áp dụng ngun lí qui nạp mạnh sau Định lí 1.2 (Nguyên lí qui nạp mạnh) Giả sử P (n) mệnh đề chứa biến tự nhiên n định nghĩa với n ≥ n0 , m số tự nhiên Khi đó, mệnh đề P (n) với số nguyên dương n ≥ n0 hai điều sau đồng thời xảy (1.2.1) Các mệnh đề P (n0 ), P (n0 + 1), , P (n0 + m) đúng, (1.2.2) Nếu mệnh đề P (n), P (n + 1), , P (n + m), n ≥ n0 mệnh đề P (n + m + 1) Chứng minh Ta chứng minh định lí phương pháp phản chứng Giả sử kết luận định lí sai, tức tồn (ít một) số n ≥ n0 mà mệnh đề P (n) sai Xét số n∗ nhỏ mà P (n∗ ) sai Khi đó, theo (1.2.1), n∗ > n0 + m, n∗ − ≥ n0 + m Mặt khác, theo tính nhỏ số n∗ , mệnh đề P (n∗ − − m), P (n∗ − − m + 1), , P (n∗ − 1) Do đó, theo (1.2.2), mệnh đề P (n∗ ) phải Điều trái với giả thiết phản chứng Vậy ta phải có P (n) với n ≥ n0 Ta dễ dàng nhận định lí 1.1 hệ định lí 1.2, cho m = Áp dụng định lí 1.2, ta có lời giải cụ thể cho ví dụ 1.3 sau Lời giải cho ví dụ 1.3 Với n = n = 2, khẳng định toán đúng, đẳng thức F2 = = 11 + 12 = F02 + F12 F4 = = 22 + 12 = F22 + F12 Giả sử khẳng định toán với n − n, với n ≥ đó, tức F2n = Fn2 + Fn−1 , 2 F2n−2 = Fn−1 + Fn−2 Ta cần chứng minh F2n+2 = Fn+1 + Fn2 Thật vậy, áp dụng hệ thức định nghĩa dãy (Fn ) đẳng thức giả thiết qui nạp, ta có F2n+2 = = = = = = 3F2n − F2n−2 2 3(Fn2 + Fn−1 ) − (Fn−1 + Fn−2 ) 2 2 2(Fn + Fn−1 ) − Fn−2 + Fn (Fn + Fn−1 )2 + (Fn − Fn−1 )2 − Fn−2 + Fn2 2 Fn+1 + Fn−2 − Fn−2 + Fn2 Fn+1 + Fn2 Vậy, theo định lí 1.2, ta có ∀n ≥ F2n = Fn2 + Fn−1 Sau ví dụ áp dụng định lí 1.2 Ví dụ 1.4 Chứng minh số nguyên dương khơng nhỏ 12 biểu diễn dạng tổng số số Trong chứng minh định lí 1.2, ta sử dụng tiên đề thứ tự để khẳng định tồn n∗ nhỏ Việc làm nầy mang tính áp đặt, nguyên lí qui nạp (đúng tiên đề qui nạp) tiên đề thứ tự tương đương Chứng minh Ta chứng minh số tự nhiên n ≥ 12 biểu diễn dạng tổng số số Với n = 12, 13, 14, 15, khẳng định đúng, 12 = + + 4, 13 = + + 5, 14 = + + 5, 15 = + + Giả sử khẳng định với số tự nhiên từ 12 đến n, n ≥ 15 Khi đó, từ n + = + (n − 3) n − biểu diễn dạng tổng số (do 12 ≤ n − ≤ n), ta suy n + biểu diễn dạng tổng số Vậy, theo định lí 1.2, ta có điều phải chứng minh Lời giải ví dụ 1.3 ví dụ 1.4 sử dụng định lí 1.2 cho trường hợp m = m = Tuy nhiên, nhiều toán, ta cần giả thiết mệnh đề P (n0 ), P (n0 + 1), , P (n) để chứng minh mệnh P (n + 1) Ta xét ví dụ sau Ví dụ 1.5 (Định lí số học) Chứng minh số tự nhiên lớn biểu diễn dạng tích số nguyên tố Phân tích - lời giải Khẳng định toán hiển nhiên với n = 2, số nguyên tố Bây ta giả sử kết luận toán với n ≥ đó, tức n phân tích thành tích số nguyên tố Ta cần chứng minh kết luận với n + Nếu n + số nguyên tố khẳng định toán hiển nhiên Khi n + hợp số, tức n + = a · b < a, b < n Do đó, giả thiết qui nạp phải số nguyên dương từ đến n − biểu diễn dạng tích số nguyên tố Lời giải cụ thể sau Khẳng định toán hiển nhiên với n = n = 3, số nguyên tố Bây ta giả sử kết luận toán với số tự nhiên từ đến n − 1, với n ≥ Ta cần chứng minh kết luận với n + Theo phân tích trên, ta cần xét trường hợp n + = a · b với < a, b < n Khi đó, theo giả thiết qui nạp, a b biểu diễn dạng tích số nguyên tố, nên tích a · b Tức khẳng định toán với n + Vậy theo định lí 1.2, ta có điều phải chứng minh Sau hai ví dụ phương pháp qui nạp (chuẩn), đòi hỏi phải có linh hoạt lập luận chứng minh Ví dụ 1.6 Chứng minh với số nguyên dương n ≥ 2, ta có bất đẳng thức 1+ 1 + + + < 23 33 n3 Chứng minh Vế trái biểu thức tăng n tăng, vế phải số, nên ta cần tìm biểu thức P (n) mà V T < P (n) < V P ∀n ≥ Bằng cách thử vài trường hợp n ta chọn P (n) = − Tuy nhiên P (2) = n 1 < + , nên ta chứng minh V T < P (n) ∀n ≥ Còn với n = 2, BĐT đề hiển nhiên Rõ ràng P (n) < ∀n ≥ 3, nên để kết thức toán, ta chứng minh 1+ 1 + + + < − 3 n n Với n = 3, BĐT trở thành + Giả sử ta có 1+ ∀n ≥ 1 + < − , hiển nhiên 27 1 + + + < − 3 n n Khi đó, 1+ 1 1 + + + + = 3 n (n + 1)3 1 1 + + + + 3 n (n + 1)3 1 < − + n (n + 1)3 1 1 = − + − + n+1 n + n (n + 1)3 n2 + n + − − = n+1 n(n + 1)3 1+ Suy ra, 1+ 1 1 + + + + < − , 23 33 n3 (n + 1)3 n+1 theo định lí 1.1, ta có 1+ 1 + + + < − 3 n n ∀n ≥ Đây điều cần chứng minh Ví dụ 1.7 (BĐT AM-GM) Cho n ≥ số thực không âm x1 , x2 , , xn Chứng minh ta ln có BĐT √ x1 + x2 + + xn ≥ n x x xn n Chứng minh Lời giải tốn chia thành ba công đoạn sau x1 + x2 a) BĐT đề với n = Thật BĐT cần chứng minh ≥ √ √ √ 22 x1 x2 , BĐT hiển nhiên đúng, tương đương với ( x1 − x2 ) ≥ 0, đẳng thức xảy x1 = x2 b) Tiếp theo, ta chứng minh, phương pháp qui nạp, BĐT đề với n số thực khơng âm với 2n số thực khơng âm Giả sử ta có BĐT √ x1 + x2 + + xn ≥ n x x xn n Xét 2n số thực không âm x1 , x2 , , xn , xn+1 , , x2n Áp dụng BĐT giả thiết qui nạp áp dụng phần a), ta có BĐT sau √ x1 + x2 + + xn ≥ n x x xn , n xn+1 + xn+2 + + x2n √ ≥ n xn+1 xn+2 x2n , n √ √ n x x x + n x √ √ n n+1 xn+2 x2n n ≥ x1 x2 xn · n xn+1 xn+2 x2n Nhân hai vế BĐT cuối với cộng theo vế với hai BĐT phía trên, ta √ x1 + x2 + + xn xn+1 + xn+2 + + x2n √ + ≥ n x1 x2 xn · n xn+1 xn+2 x2n n n x1 + x2 + + xn + xn+1 + + x2n √ ≥ 2n x1 x2 xn xn+1 x2n ⇐⇒ 2n c) Tiếp theo ta chứng minh BĐT đề với n số thực không âm với n − số thực không âm (qui nạp lùi) Thật vậy, giả sử ta có √ x1 + x2 + + xn ≥ n x x xn n x1 + x2 + + xn−1 Khi đó, thay xn = ta n−1 ⇐⇒ ⇐⇒ ⇐⇒ ⇐⇒ + +xn−1 x1 + x2 + + xn−1 + x1 +x2n−1 n x1 + x2 + + xn−1 n−1 n x1 + x2 + + xn−1 n−1 n−1 x1 + x2 + + xn−1 n−1 x1 + x2 + + xn−1 n−1 tức BĐT đề với n − Kết hợp phần trên, ta suy ≥ n + +xn−1 x1 x2 xn−1 x1 +x2n−1 ≥ n + +xn−1 x1 x2 xn−1 x1 +x2n−1 ≥ + +xn−1 x1 x2 xn−1 x1 +x2n−1 ≥ x1 x2 xn−1 ≥ √ n−1 x1 x2 xn−1 , +) BĐT đề với n = (theo a)), +) Nếu BĐT đề với n ≥ với 2n (theo b)), với 2n − 1, 2n − 2, , n + (theo c)) Vậy theo định lí 1.1, ta có BĐT đề với n nguyên dương ≥ 2.2 Để kết thúc phần này, ta xét thêm số ví dụ sau Ví dụ 1.8 Chứng minh số tập tập n phần tử 2n Chứng minh Bước sở dễ thấy, tập có phần tử (tập rỗng) có tập (cũng tập rỗng) Bước qui nạp Giả sử số tập tập n phần tử 2n Xét tập n + phần tử bất kì, chẳng hạn X = {x1 , x2 , , xn , xn+1 } Ta chứng minh X có 2n+1 tập Ta chia tập X thành hai loại: loại I gồm tập có chứa xn+1 loại II gồm tập lại Loại II gồm tập tập n phần tử Y = {x1 , x2 , , xn }, theo giả thiết qui nạp, số tập loại 2n Với tập loại I, ta bỏ phần tử xn+1 , tập tập Y (và ngược lại, với tập Y , ta thêm vào phần tử xn+1 trở thành tập loại I) Do đó, số tập loại I với số tập tập n phần tử Y Suy số tập X 2n + 2n = 2n+1 khẳng định Vậy theo ngun lí nói định lí 1.1, ta có điều phải chứng minh Ví dụ 1.9 (Trò chơi tháp Hà Nội) Ta xét tốn trò chơi tháp Hà Nội sau Ban đầu cho n đĩa đường kính 1, 2, , n cọc (các đĩa có lỗ tâm để xâu vào cọc) Ban đầu, đĩa xếp cọc (gọi cọc nguồn N) theo thứ tự đường kính giảm dần tính từ lên Mỗi lượt đi, người chơi chuyển đĩa cọc nguồn sang hai cọc lại (gọi cọc trung gian T cọc đích Đ) cho q trình chuyển đĩa, đĩa có đường kính lớn khơng xếp đĩa có đường kính nhỏ Người chơi coi thắng cuộc, chuyển tất đĩa sang cọc Đ, tất nhiên bước phải luật chơi nêu a) Hãy tìm thuật chơi để chiến thắng b) Chứng minh có thuật chơi để chiến thắng sau 2n − lượt 2n − số lượt lí tưởng Lời giải a) Trường hợp n = 1, ta chẳng có đáng để nói Nếu có thuật chơi chiến thắng cho trường hợp n đĩa, cọc (N, T, Đ), có thuật chơi chiến thắng cho trường hợp n + đĩa Thật vậy, ta coi đĩa đường kính n + (gọi tắt đĩa n + 1) gắn chặt vào sàn cọc Trước hết ta thực thuật toán chiến thắng với n đĩa xếp đĩa n + 1, cọc (N, Đ, T) (ở ta đổi vai trò hai cọc T Đ cho nhau) Kết thúc thuật toán này, Phương pháp chứng minh qui nạp ví dụ gọi phương pháp qui nạp tiến - lùi (hay qui nạp Cauchy) Bài toán chứng minh BĐT AM-GM ví dụ điển hình cho phương pháp đĩa 1, 2, , n chuyển sang cọc T Tiếp theo ta chuyển đĩa n + sang cọc Đ thực thuật toán chiến thắng với n đĩa cọc T cọc (T, N, Đ) b) Ta kí hiệu Tn số bước tối thiểu để chiến thắng trường hợp n đĩa Ta chứng minh Tn = 2n − Bước sở hiển nhiên Bước qui nạp, theo phần a), tồn thuật toán chiến thắng cho trường hợp n + đĩa với Tn + + Tn = 2Tn + = 2n+1 − nước Hơn nữa, muốn chuyển đĩa n + sang cọc Đ phải chuyển n đĩa sang cọc khác, phải chuyển n đĩa lại cọc Đ (xếp đĩa n + 1), với bước chuyển đĩa n + 1, ta suy thuật toán chiến thắng với n + đĩa phải bao gồm 2n+1 − bước Vậy Tn+1 = 2n+1 − Ví dụ 1.10 (IMO2011) Giả sử n > số nguyên Cho cân hai đĩa n cân với trọng lượng 20 , 21 , , 2n−1 Ta muốn đặt lên cân n cân, một, theo cách để bảo đảm đĩa cân bên phải không nặng đĩa cân bên trái Ở bước ta chọn cân chưa đặt lên cân, đặt vào đĩa bên trái, vào đĩa bên phải, tất cân đặt lên cân Xác định xem có cách để thực mục đích đề Lời giải Ta kí hiệu fn số cách để thực mục đích đề Khi n = 1, ta có cân, nên có cách thực đặt cân lên đĩa cân bên trái, hay f1 = Với n = 2, ta có hai cân Ta có cách thực sau: 1T2T (đặt cân lên đĩa cân bên trái, đặt cân lên đĩa bên trái), 2T1T, 2T1P Suy f2 = Nếu tiếp tục với n = 3, ta đếm f3 = 15 Nhưng với n = 4, số cách thực nhiều, việc ngồi liệt kê cách thực không hiệu Hơn nữa, từ kết đầu ta chưa dự đốn cơng thức cho fn Bởi ta xây dựng mối quan hệ truy hồi cho fn+1 theo fn trước Xét cách thực mục đích đề với n + cân 1, 2, 22 , , 2n Để ý ngoại trừ cân 1, cân lại có trọng lượng sai khác nên cân khơng đặt đặt tùy ý vào hai đĩa cân mà khơng ảnh hưởng đến mục đích đề Như vậy, với cách thực với n cân 2, 22 , , 2n , ta có 2(n − 1) + = 2n − cách ghép thêm cân để đạt mục đích (nếu đặt cân bước có cách, đặt bước sau có hai cách bước) Việc lại ta đếm số cách thực với n cân 2, 22 , , 2n để đạt mục đích Để ý chia trọng lượng cân cho số cách thực không đổi Vậy số cách thực với n cân fn Từ lập luận ta có fn+1 = (2n − 1) · fn Đến đây, với giá trị f1 , f2 , f3 , ta suy công thức cho fn fn = (2n − 1)!! = · · · (2n − 1) Lập luận chứng minh công thức Ví dụ 1.11 Ban đầu bảng viết số Ta viết thêm số n lên bảng, n thu từ số k có bảng theo qui luật sau: (a) n thu cách viết thêm chữ số vào tận bên phải k, (b) n thu cách viết thêm chữ số vào tận bên phải k, (c) n thu cách chia k cho k chẵn Hỏi sau số hữu hạn lần thực thao tác trên, thu số ngun dương n hay khơng? Lời giải Câu trả lời khẳng định Ta chứng minh phương pháp qui nạp mạnh Bước sở Ta viết số từ đên 10, mô tả (c) (c) −→ −→ , (b) (c) (c) (c) −→ 24 −→ 12 −→ −→ , (a) (c) −→ 10 −→ , (b) (c) (c) (c) −→ 14 −→ , (b) (c) −→ 64 −→ 32 −→ 16 −→ , (b) (b) (c) (c) (c) (c) −→ 14 −→ 144 −→ 72 −→ 36 −→ 18 −→ Bây giả sử ta viết số từ đến n ≥ 10 Xét số n + 1, ta chia thành trường hợp sau • n + ≡ (hoặc 4) (mod 10) Lúc này, số n+1 10 tương ứng (n + 1) − 10 n+1 < n Do đó, 10 ta viết số n + cách viết thêm số (hoặc 4) vào tận bên n+1 (n + 1) − phải số (tương ứng ) 10 10 viết lên bảng, theo giả thiết qui nạp, ≤ • n + = 10k + r với r ∈ {1, 2, 3, 5, 6, 7, 8}, ta viết số n + tương tự trường hợp Cụ thể bảng sau 10 (b) (c) (c) 10k + 4k −→ 40k + −→ 20k + −→ 10k + 10k + 2k −→ 20k + −→ 10k + 10k + 8k + −→ 80k + 24 −→ 40k + 12 −→ 20k + −→ 10k + 10k + 2k + −→ 20k + 10 −→ 10k + 10k + 4k + −→ 40k + 24 −→ 20k + 12 −→ 10k + 10k + 2k + −→ 20k + 14 −→ 10k + 10k + 8k + −→ 80k + 64 −→ 40k + 32 −→ 20k + 16 −→ 10k + (b) (b) (c) (c) (c) (a) (c) (c) (b) (c) (b) (c) (b) (c) (c) (c) (c) • n + = 10k + Trước hết, bảng có số 16k + 14 ta viết số 10k + theo sơ đồ sau (b) (c) (c) (c) (c) 16k + 14 −→ 160k + 144 −→ 80k + 72 −→ 40k + 36 −→ 20k + 18 −→ 10k + Do vậy, ta cần tìm cách viết số m = 16k + 14 trước Để ý m số chẵn, nên theo hai trường hợp trên, m (mod 10) ∈ {0, 2, 4, 6} ta thu số m từ số l thuộc tập 2k 4k + m + (m + 1) − , , m, m 10 10 10k + 10k + 4k + m < m = 8k + < 10k + 10k + Còn m ≡ (mod 10), theo trên, ta thu số m từ số m1 8k + m1 chẵn (chưa có bảng) mà ≤ < Lặp lại m 10k + lập luận với m m1 , tình "xấu nhất", m1 thu từ m2 , m2 thu từ m3 Lúc này, Lúc l có bảng, l ≤ m3 < m < 10k + 9, nên m3 viết bảng Nên ta viết số m2 , m1 , m = 16k + 14 n + = 10k + Bước qui nạp kết thúc Ví dụ 1.12 (Định lí biểu diễn g−phân) Cho số nguyên dương g ≥ Chứng minh với số nguyên dương n biểu diễn dạng n = ak g k + ak−1 ak−1 + + a2 g + a1 g + a0 , k số ngun khơng âm thỏa mãn g k ≤ n < g k+1 , số a0 , a1 , , ak thuộc tập {0, 1, 2, , g − 1} ak = 11 Biểu diễn gọi biểu g−phân n hay biểu diễn n theo số g Ta thường viết n = bk bk−1 · · · b0 (g) thay cho cách biểu diễn trên, số a0 , a1 , , ak gọi chữ số n hệ số g Khi g = 10, cách viết cách viết số n hệ thập phân Chứng minh Ta chứng minh phương pháp qui nạp mạnh theo k, với k nguyên không âm, số tập {g k , g k + 1, , g k+1 − 1} có biểu diễn g−phân Phần chứng minh tính biểu diễn xin dành cho bạn đọc Với k = 0, khẳng định hiển nhiên, số nguyên dương từ đến g − biểu diễn g−phân Giả sử khẳng định đến k − ≥ 0, ta chứng minh với k Xét số nguyên n mà g k ≤ n < g k+1 − Bằng thuật toán chia, ta viết n dạng n = ak g k + r, với ≤ r < g k Trước hết ta ≤ ak ≤ g − Thật vậy, ak = n = r < g k ≤ n, vơ lí Còn ak ≥ g n = g k+1 + r ≥ g k+1 > n, vơ lí Nếu r = n = ak g k biểu diễn g−phân n, với ≤ ak ≤ g − 1, a0 = a1 = = ak−1 = Nếu ≤ r < g k tồn số nguyên không âm l ≤ k − cho g l ≤ r < g l+1 , lúc theo giả thiết quy nạp, r có biểu diễn g−phân dạng r = al g l + al−1 g l−1 + + a1 g + a0 Suy n có biểu diễn g−phân sau n = ak g k + ak−1 g k−1 + + al+1 g l+1 + al g l + al−1 g l−1 + + a1 g + a0 , đó, ta chọn ak−1 = ak−2 = = al+1 = l < k − Qua ví dụ trên, ta khẳng định lại phương pháp chứng minh qui nạp quan trọng tốn học Các ví dụ nêu chưa nhiều (đặc biệt dạng toán) nói lên đa dạng phạm vi áp dụng giải toán phương pháp Nội dung phương pháp đơn giản cần ý đến "kĩ thuật" chứng minh qui nạp: kĩ thuật làm trội (làm giảm), kĩ thuật qui nạp Cauchy, kĩ thuật qui nạp song song, kĩ thuật xây dựng cấu hình qui nạp 12 Bài tập Bài 2.1 Chứng minh với số nguyên dương n, ta có đẳng thức 1 1 1 + + + = − + − + − n+1 n+2 2n 2n − 2n Bài 2.2 Chứng minh với n nguyên dương, ta có 13 + 23 + + n3 = (1 + + + n)2 Bài 2.3 Chứng minh với n nguyên dương, ta có a) 32n+3 + 40n − 27 64, b) 52n−1 2n + 3n+1 22n−2 19, c) 32 4n+1 + 11, d) 3(15 + 25 + + n5 ) 13 + 23 + + n3 Bài 2.4 Gọi x1 , x2 hai nghiệm phương trình x2 − 6x + = Chứng minh với n nguyên, số sn = xn1 + xn2 số nguyên không chia hết cho Bài 2.5 Cho k số nguyên dương lẻ Chứng minh với n nguyên dương ta có n k − 2n+2 Bài 2.6 Với số nguyên dương n, gọi an , bn , cn số nguyên cho n √ √ √ 3 − = an + bn + cn Chứng minh cn ≡ (mod 3) n ≡ (mod 3) Bài 2.7 Chứng minh số hữu tỉ dương viết dạng thương tích giai thừa số nguyên tố (không thiết phân 10 2! · 5! biệt), chẳng hạn = 3! · 3! · 3! Bài 2.8 Cho dãy Fibonacci F0 = F1 = 1, Fn+2 = Fn+1 + Fn ∀n ≥ Chứng minh a) F02 + F12 + + Fn2 = Fn Fn+1 b) Fn+1 Fn−1 − Fn2 = (−1)n+1 ∀n ≥ 0, ∀n ≥ 1, 13 c) Fn+1 = Fk+1 Fn−k + Fk Fn−k−1 ∀0 ≤ k < n Bài 2.9 Chứng minh với k nguyên dương ≥ 3, ta biểu diễn thành tổng nghịch đảo k số nguyên dương phân biệt Bài 2.10 (BĐT Bec-nu-li) Chứng minh với n nguyên dương với x ≥ −1 ta có BĐT (1 + x)n ≥ + nx Bài 2.11 Chứng minh với n nguyên dương ta có BĐT 1+ n n < Bài 2.12 Cho n số nguyên dương ≥ số thực dương a1 , a2 , , an lớn Chứng minh ta ln có BĐT 1 n + + + ≥ √ n + a1 + a2 + an + a1 a2 an Bài 2.13 (VMO2011) Chứng minh x > n ∈ N>0 , ta có bất đẳng thức 2n+1 x+1 xn (xn+1 + 1) ≤ xn + Bài 2.14 Trong mặt phẳng cho số đường thẳng phân biệt Chúng chia mặt phẳng thành số miền Chứng minh ta tơ miền miền hai màu đen, trắng cho miền tô hai miền kề tơ màu khác Bài 2.15 Cho V đa giác lồi a) Chứng minh V có 3k đỉnh V tam giác hóa cho với đỉnh V , số tam giác nhận làm (một) đỉnh số lẻ b) Chứng minh V có 3k + đỉnh V tam giác hóa cho có hai đỉnh V mà với đỉnh số đó, số tam giác nhận làm (một) đỉnh số chẵn (Một cách tam giác hóa đa giác hiểu chia đa giác thành tam giác đường chéo không cắt nó.) Bài 2.16 (IMO2013) Chứng minh với hai số nguyên dương túy ý k n, tồn k số nguyên dương m1 , m2 , , mk (không thiết phân biệt) cho 1 2k − = 1+ 1+ + 1+ n m1 m2 mk Bài 2.17 (Bulgari1996) Chứng minh với n nguyên dương ≥ 3, tồn số nguyên dương lẻ (xn , yn ) cho 7x2n + yn2 = 2n 14 Bài 2.18 (VMO2010) Chứng minh với n nguyên dương, phương trình x2 + 15y = 4n có n nghiệm ngun (x, y) Bài 2.19 Cho n số nguyên dương n số nguyên a1 , a2 , , an có ước chung lớn Tập S gồm số nguyên thỏa mãn đồng thời điều kiện sau i) ∈ S ∀i ∈ {1, 2, , n}, ii) − aj ∈ S ∀i, j ∈ {1, 2, , n}, iii) ∀x, y ∈ S, x + y ∈ S x − y ∈ S Chứng minh S = Z Bài 2.20 (IMO2004) Cho n ≥ số nguyên Gọi t1 , t2 , , tn số thực dương thỏa mãn n2 + > (t1 + t2 + + tn ) 1 + + + t1 t2 tn Chứng minh ti , tj , tk cạnh tam giác, với i, j, k mà ≤ i < j < k ≤ n 15 Lời giải - Hướng dẫn Bài 2.1 Hướng dẫn Bước qui nạp: 1− = 1 1 1 + − + − + − 2n − 2n 2n + 2n + 1 1 1 + − + − + − = 2n − 2n 2n + 2n + 1 1 + + + + − = n+1 n+2 2n 2n + 2n + 1 1 = + + + + − n+2 2n 2n + n + 2n + 1 1 = + + + + n+2 2n 2n + 2n + 1− Bài 2.2 Hướng dẫn Ta có + + + n = n(n + 1) Bước qui nạp chứng minh đẳng thức 13 + 23 + + n3 + (n + 1)3 = (13 + 23 + + n3 ) + (n + 1)3 = n(n + 1) 2 + (n + 1)3 = (n + 1)(n + 2) 2 Bài 2.3 Hướng dẫn a) 32(n+1)+3 + 40(n + 1) − 27 = (32n+3 + 40n − 27) − 320n + 256 b) 52(n+1)−1 2n+1 + 3(n+1)+1 22(n+1)−2 = 50 (52n−1 2n + 3n+1 22n−2 ) − 38 · 3n+1 22n−2 4(n+1)+1 c) 32 4n+1 +2 = 32 16 −216 +216 +2 = 4n+1 32 16 − 216 +2 (25 ) + Để ý a2n − b2n chia hết cho a + b d) Chứng minh 15 + 25 + + n5 = n2 (n + 1)2 (2n2 + 2n − 1) ∀n ≥ sử dụng 12 kết 2.2 Bài 2.4 Hướng dẫn Ta cần chứng minh khẳng định tốn với n ngun −n n n khơng âm, x−n = x2 , x2 = x1 Để ý ta có sn+2 = 6sn+1 − sn ∀n ≥ 0, nên với s0 = 2, s1 = 6, qui nạp, ta suy sn ∈ Z∀n ≥ Để chứng minh 16 sn không chia hết cho 5, ý sn+2 = 6sn+1 − sn = (6sn − sn−1 ) − sn = 35sn − 6sn−1 ∀n ≥ n+1 Bài 2.5 Hướng dẫn Để ý k n với k lẻ k + số chẵn − = k2 n n n − = (k − 1)(k + 1) Bài 2.6 Hướng dẫn Trước hết chứng minh x, y, z ∈ Z √ √ 3 x + y + z = ⇐⇒ x = y = z = Ta lại có √ √ an+3 + bn+3 + cn+3 = = = = √ (√ − 1)n+3√ ( − 1)√n ( −√ 1)3 √ √ (an + bn + cn 4)(7 + 3 − 3 4) √ (7an − 6bn + 6cn ) + (3an + 7bn −√6cn ) +(−3an + 3bn + 7cn ) 4, suy cn+3 = −3an + 3bn + 7cn ≡ cn (mod 3) Việc lại kiểm tra c1 , c2 , c3 thấy có c2 ≡ (mod 3) Bài 2.7 Hướng dẫn Ta sử dụng nguyênquy nạp mạnh Giả sử kết luận toán đến n, với n + 1, ta xét hai trường hợp Nếu n + hợp số kết luận theo giả thiết qui nạp, n + số nguyên tố sử dụng phân (n + 1)! , n! tích số khơng vượt q n nên theo giả tích n + = n! thiết qui nạp n! viết dạng thương tích giai thừa số nguyên tố Bài 2.8 Hướng dẫn 2 a) F02 + F12 + + Fn2 + Fn+1 = (F02 + F12 + + Fn2 ) + Fn+1 = Fn Fn+1 + Fn+1 = Fn+1 Fn+2 b) Chứng minh Fn+2 Fn − Fn+1 + (Fn+1 Fn−1 − Fn2 ) = c) Sử dụng nguyên lí qui nạp mạnh Do Fn+2 = Fn+1 + Fn nên có Fn = Fk+1 Fn−1−k + Fk Fn−k−2 ∀0 ≤ k < n − Fn+1 = Fl+1 Fn−l + Fl Fn−l−1 ∀0 ≤ l < n chọn l = k < n − ta có Fn+2 = Fk+1 Fn−1−k + Fk Fn−k−2 + Fk+1 Fn−k + Fk Fn−k−1 17 ∀0 ≤ k < n − = (Fk+1 Fn−1−k + Fk+1 Fn−k ) + (Fk Fn−k−2 + Fk Fn−k−1 ) ∀0 ≤ k < n − ∀0 ≤ k < n − = Fk+1 Fn+1−k + Fk Fn−k Trường hợp k = n − k = n, ta có đẳng thức sau Fn+2 = Fn F2 + Fn−1 F1 , Fn+2 = Fn+1 F1 + Fn F0 Bài 2.9 Hướng dẫn Với k = khẳng định toán đúng, = 1 + + Giả sử khẳng định với k 1= 1 + + + , n1 n2 nk n1 < n2 < < nk Khi đó, sử dụng đẳng thức 1 = + nk nk + nk (nk + 1) ta suy khẳng định toán với k + Bài 2.10 Hướng dẫn Ta chứng minh phương pháp qui nạp theo n Bước qui nạp để ý có (1 + x)n ≥ + nx (1 + x)n+1 = (1 + x)n (1 + x) ≥ (1 + nx)(1 + x) = + (n + 1)x + nx2 ≥ + (1 + n)x Bài 2.11 Hướng dẫn Trước hết, ta có 1+ Tiếp theo chứng minh dãy an = 1+ n Dẫn đến n n < 1+ n n+1 ≤ 1+ n n 1+ n+1 n+1 1+ dãy giảm, suy < ∀n ≥ n < ∀n ≥ Với giá trị n = 1, 2, 3, 4, 5, cách thử trực tiếp, ta thấy BĐT đề 18 Bài 2.12 Hướng dẫn Sử dụng phương pháp qui nạp Cauchy ví dụ 1.7 Bài 2.13 Lời giải Ta chứng minh BĐT đề phương pháp qui nạp theo n x+1 x(x2 + 1) ≤ Với n = ta cần chứng minh BĐT tương đương với x+1 ⇐⇒ ⇐⇒ ⇐⇒ ⇐⇒ ⇐⇒ ⇐⇒ 8x(x2 + 1) 8x(x2 + 1) 8x(x2 + 1) 8x(x2 + 1) 0 ≤ (x + 1)4 ≤ (x2 + 2x + 1)2 ≤ ((x2 + 1) + 2x) ≤ (x2 + 1)2 + 4x(x2 + 1) + 4x2 ≤ (x2 + 1)2 − 4x(x2 + 1) + 4x2 ≤ (x2 + − 2x)2 ≤ (x − 1)4 , BĐT hiển nhiên đúng, đẳng thức xảy x = Bây giả sử với n ≥ ta có xn (xn+1 + 1) ≤ xn + 2n+1 x+1 (1) Ta cần chứng minh xn+1 (xn+2 + 1) ≤ xn+1 + Nhân hai vế (1) với x+1 2 xn (xn+1 + 1) xn + x+1 2n+3 (2) > 0, ta có x+1 ≤ x+1 2n+3 Do đó, để chứng minh (2), ta cần chứng minh xn (xn+1 + 1) xn + x+1 ≥ xn+1 (xn+2 + 1) xn+1 + BĐT tương đương với ⇐⇒ ⇐⇒ ⇐⇒ ⇐⇒ xn (xn+1 + 1)(x + 1)2 4(xn + 1) n+1 + 1)2 (x + 1)2 (x 2n+2 (x + 2xn+1 + 1)(x + 1)2 ((x2n+2 + 1) + 2xn+1 ) (x + 1)2 ((x2n+2 + 1) + 2xn+1 ) (x + 1)2 ≥ ≥ ≥ ≥ ≥ 19 xn+1 (xn+2 + 1) xn+1 + n+2 4x(x + 1)(xn + 1) 2n+2 4x(x + xn+2 + xn + 1) 4x(x2n+2 + 1) + 4x(xn+2 + xn ) 4x(x2n+2 + 1) + 4x(xn+2 + xn ) ⇐⇒ ⇐⇒ ⇐⇒ ⇐⇒ ⇐⇒ ⇐⇒ (x2n+2 + 1)(x + 1)2 + 2xn+1 (x + 1)2 (x2n+2 + 1)(x + 1)2 − 4x(x2n+2 + 1) (x2n+2 + 1) ((x + 1)2 − 4x) (x2n+2 + 1)(x − 1)2 (x − 1)2 (x2n+2 + − 2xn+1 ) (x − 1)2 (xn+1 − 1)2 ≥ 4x(x2n+2 + 1) + 4xn+1 (x2 + 1) ≥ 4xn+1 (x2 + 1) − 2xn+1 (x + 1)2 ≥ 2xn+1 (2(x2 + 1) − (x + 1)2 ) ≥ 2xn+1 (x − 1)2 ≥ ≥ BĐT cuối rõ ràng đúng, đẳng thức xảy x = Ta có điều phải chứng minh, theo nguyên lí qui nạp Bài 2.14 Hướng dẫn Trường hợp có n = đường thẳng khơng có đáng bàn Giả sử với n đường thẳng ta tô màu miền thỏa mãn yêu cầu tốn Xét trường hợp có n + đường thẳng l1 , l2 , , ln , ln+1 Coi ln+1 "ẩn đi", tô màu miền n đường thẳng l1 , l2 , , ln tạo Việc làm theo giả thiết qui nạp Trở lại với đường thẳng ln+1 , chia mặt phẳng thành hai nữa, ta giữ nguyên màu miền nửa mặt phẳng đó, nửa mặt phẳng lại, ta đổi màu miền từ trắng thành đen từ đen thành trắng Dễ thấy cách tô màu thỏa mãn yêu cầu đề Bài 2.15 Hướng dẫn a) Giả sử ta tam giác hóa theo u cầu tốn đa giác có 3k đỉnh Xét đa giác có 3k + đỉnh A1 A2 A3k+3 Ta nối A1 với A3k , theo giả thiết qui nạp, đa giác A1 A2 A3k tam giác hóa thỏa mãn yêu cầu toán Xét hệ gồm đường chéo tam giác hóa đa giác A1 A2 A3k với đường chéo A1 A3k+2 , A3k A3k+2 , hệ đường chéo tam giác hóa đa giác A1 A2 A3k+3 thỏa mãn yêu cầu b) Có thể dùng kết câu a) qui nạp trực tiếp Bài 2.16 Hướng dẫn Sử dụng đẳng thức 1+ 1+ 2k+1 − = n 2k+1 − = n 1+ 1+ 2k − (n + 1)/2 2k − n/2 1+ 1+ n với n lẻ, n+ 2k+1 −2 với n chẵn Bài 2.17 Lời giải Với n = 3, ta chọn x3 = y3 = Giả sử với n ≥ 3, ta tìm số nguyên lẻ (xn , yn ) cho 7x2n + yn2 = 2n Ta cần tìm số nguyên lẻ (x, y) cho 7x2 + y = 2n+1 20 Ta có xn + yn 7 + xn − y n 2 7xn − yn 2 = 2(7xn + yn )2 = 2n+1 , 7xn + yn + = 2(7xn + yn )2 = 2n+1 |xn − yn | xn + y n |xn − yn | xn + y n khác tính chẵn lẻ + = 2 2 xn + yn |7xn − yn | |xn − yn | max{xn , yn } số lẻ Ngoài ra, cặp số , 2 7xn + yn xn + yn tính chẵn lẻ Nên lẻ, ta chọn 2 Để ý (x, y) = xn + yn |7xn − yn | , 2 , |xn − yn | 7xn + yn , 2 |xn − yn | lẻ, ta chọn (x, y) = Bài 2.18 Hướng dẫn Để ý (x, y) nghiệm phương trình x2 + 15y = 4n (2x, 2y) nghiệm phương trình x2 + 15y = 4n+1 Do đó, ta cần chứng minh với n nguyên dương, phương trình x2 + 15y = 4n có nghiệm (x, y) mà x, y lẻ đủ Việc tương tự 2.17, với việc sử dụng đẳng thức x + 15y 2 x − 15y 2 x−y 2 + 15 + 15 x+y = 4(x2 + 15y ), = 4(x2 + 15y ) Bài 2.19 Trước hết, ∈ S, theo ii), = a1 − a1 ∈ S Nếu có x ∈ S theo iii), + x = x ∈ S nên −x = − x ∈ S Nếu có x ∈ S theo trên, −x ∈ S, theo iii), x + (−x) = ∈ S nên x − (−x) = 2x ∈ S, −2x ∈ S Tương tự có 0, ±x, ±2x, , ±lx ∈ S với l ≥ từ x+(−lx) = −(l −1)x ∈ S nên theo iii), x+lx = x−(−lx) ∈ S Suy ±(l + 1)x ∈ S Vậy theo nguyên lí qui nạp mạnh, nx ∈ S ∀n ∈ Z Như vậy, ta cần chứng minh ∈ S Rõ ràng tồn số số a1 , a2 , , an , giả sử a b (với a ≤ b) mà ước chung lớn a b Khi đó, tồn số nguyên x, y cho 21 xb + ya = Theo chứng minh trên, na ∈ S ∀n ∈ Z, nb ∈ S ±xb, ±ya ∈ S, theo ii), = xb + ya = xb − (−ya) ∈ S ∀n ∈ Z Suy Bài 2.20 Không tổng quát, ta giả sử t1 ≤ t2 ≤ ≤ tn Ta cần chứng ti + tj > tk ∀1 ≤ i < j < k ≤ n Với n = 3, giả sử t3 ≥ t1 + t2 , với giả thiết, ta suy 10 > = ≥ = ≥ 1 + + t1 t2 t3 t1 + t2 1 t1 t2 3+ + t3 + + + t3 t1 t2 t2 t1 t1 + t2 3+ + t3 +2 t3 t1 + t2 t1 + t2 t3 t3 5+ + +3 t3 t1 + t2 t1 + t2 + + 3, (t1 + t2 + t3 ) vơ lí Vậy t3 < t1 + t2 Giả sử kết luận toán với n, ta chứng minh với n + Ta cần chứng minh tn+1 < t1 + t2 đủ Thật vậy, từ giả thiết ta có (n + 1)2 + > (t1 + t2 + + tn + tn+1 ) 1 1 + + + + t1 t2 tn tn+1 1 1 + + + + +1 t1 t2 tn−1 tn+1 t1 + t2 + + tn−1 + tn+1 1 1 + + tn + + + + tn t1 t2 tn−1 tn+1 1 1 ≥ (t1 + t2 + + tn−1 + tn+1 ) +1 + + + + t1 t2 tn−1 tn+1 t1 + t2 + + tn−1 + tn+1 n2 + + tn tn t1 + t2 + + tn−1 + tn+1 1 1 ≥ (t1 + t2 + + tn−1 + tn+1 ) + + + + + + 2n t1 t2 tn−1 tn+1 = (t1 + t2 + + tn−1 + tn+1 ) Suy n2 > (t1 + t2 + + tn−1 + tn+1 ) 1 1 + + + + t1 t2 tn−1 tn+1 , theo giả thiết qui nạp, số n số t1 , t2 , , tn−1 , tn+1 cạnh tam giác, suy tn+1 < t1 + t2 , ta có điều cần chứng minh 22 ... = ak−1 = Nếu ≤ r < g k tồn số nguyên không âm l ≤ k − cho g l ≤ r < g l+1 , lúc theo giả thiết quy nạp, r có biểu diễn g−phân dạng r = al g l + al−1 g l−1 + + a1 g + a0 Suy n có biểu diễn... (mod 3) Việc lại kiểm tra c1 , c2 , c3 thấy có c2 ≡ (mod 3) Bài 2.7 Hướng dẫn Ta sử dụng nguyên lí quy nạp mạnh Giả sử kết luận toán đến n, với n + 1, ta xét hai trường hợp Nếu n + hợp số kết luận

Ngày đăng: 03/05/2018, 12:35

Xem thêm:

TỪ KHÓA LIÊN QUAN

w